2
$\begingroup$

Let $m_i\geq 2 (1\leq i\leq n)$ be $n$ pairwisely coprime positive integers and let $q_i\geq 2 (1\leq i\leq n)$ be $n$ arbitrary prime powers, let$A=\prod_{i=1}^n(({q_i}^{m_i}-1)/(q_i-1))$. Let $\sigma(A)$ be the number of different prime factors of A, is it true that $\sigma(A)\geq n$? If this is not true, is there a counterexample? Is there a good way to estimate $\sigma(A)$?

$\endgroup$
3
  • 8
    $\begingroup$ Bad idea to use $\sigma(A)$ for the number of prime factors of $A$ when 1) everyone else uses $\sigma$ for the sum of the divisors and 2) everyone else uses $\omega$ for the number of prime factors. $\endgroup$ Dec 20, 2010 at 21:48
  • $\begingroup$ If the q_i are all equal this should follow, at least modulo a few exceptions, from Zsigmondy's theorem: en.wikipedia.org/wiki/Zsigmondy's_theorem $\endgroup$ Jan 4, 2011 at 11:38
  • $\begingroup$ @Qiaochu: You don't need them to be equal, you just need them to be powers of the same prime. Also Zsigmondy's theorem helps when $m_i$'s have lots of divisors. $\endgroup$ Jan 4, 2011 at 12:04

2 Answers 2

10
$\begingroup$

No it's not true you have the following counterexample: $$ \frac{2^5-1}{2-1} \times \frac{5^3-1}{5-1} = 31^2 $$

$\endgroup$
1
$\begingroup$

Let take a look to the special case when your $q_i$ are actually $n$ distinct \emph{odd} prime numbers.

I use the standard notations : $\omega(H)$ is the number of distinct prime divisors of $H$ and $\sigma(G)$ is the sum of all positive divisors of $G.$

Put $B$ the product of all the $q_i^{m_i}$

then we have

$$ \sigma(B) = mB $$

if $B$ is an odd $m$-multi-perfect number.

((sure, we do not known concrete examples of this, but...)

So, in this case

$$ \omega(B) = n $$

and you have your lower bound attained.

luis


$\endgroup$
2
  • $\begingroup$ However, the question asks about $\omega(\sigma(B)) = \omega(mB)$, not $\omega(B)$. $\endgroup$ Sep 4, 2011 at 5:43
  • $\begingroup$ warn: Huan used sigma'' to denote omega''. Anyway what do you think is the interesting question here ?. No feedback seems known from the OP... $\endgroup$ Sep 6, 2011 at 18:06

Your Answer

By clicking “Post Your Answer”, you agree to our terms of service and acknowledge you have read our privacy policy.

Not the answer you're looking for? Browse other questions tagged or ask your own question.